Back Home

  • Science Notes Posts
  • Contact Science Notes
  • Todd Helmenstine Biography
  • Anne Helmenstine Biography
  • Free Printable Periodic Tables (PDF and PNG)
  • Periodic Table Wallpapers
  • Interactive Periodic Table
  • Periodic Table Posters
  • How to Grow Crystals
  • Chemistry Projects
  • Fire and Flames Projects
  • Holiday Science
  • Chemistry Problems With Answers
  • Physics Problems
  • Unit Conversion Example Problems
  • Chemistry Worksheets
  • Biology Worksheets
  • Periodic Table Worksheets
  • Physical Science Worksheets
  • Science Lab Worksheets
  • My Amazon Books

Potential And Kinetic Energy Example Problem – Work and Energy Examples

Potential energy is energy attributed to an object by virtue of its position. When the position is changed, the total energy remains unchanged but some potential energy gets converted into kinetic energy . The frictionless roller coaster is a classic potential and kinetic energy example problem.

The roller coaster problem shows how to use the conservation of energy to find the velocity or position or a cart on a frictionless track with different heights. The total energy of the cart is expressed as a sum of its gravitational potential energy and kinetic energy. This total energy remains constant across the length of the track.

Potential And Kinetic Energy Example Problem

Rollercoaster Diagram for Conservation of Energy Example Problem

A cart travels along a frictionless roller coaster track. At point A, the cart is 10 m above the ground and traveling at 2 m/s. A) What is the velocity at point B when the cart reaches the ground? B) What is the velocity of the cart at point C when the cart reaches a height of 3 m? C) What is the maximum height the cart can reach before the cart stops?

The total energy of the cart is expressed by the sum of its potential energy and its kinetic energy.

Potential energy of an object in a gravitational field is expressed by the formula

where PE is the potential energy m is the mass of the object g is the acceleration due to gravity = 9.8 m/s 2 h is the height above the measured surface.

Kinetic energy is the energy of the object in motion. It is expressed by the formula

KE = ½mv 2

where KE is the kinetic energy m is the mass of the object v is the velocity of the object.

The total energy of the system is conserved at any point of the system. The total energy is the sum of the potential energy and the kinetic energy.

Total E = KE + PE

To find the velocity or position, we need to find this total energy. At point A, we know both the velocity and the position of the cart.

Total E = KE + PE Total E = ½mv 2  + mgh Total E = ½m(2 m/s) 2  + m(9.8 m/s 2 )(10 m) Total E = ½m(4 m 2 /s 2 ) + m(98 m 2 /s 2 ) Total E = m(2 m 2 /s 2 ) + m(98 m 2 /s 2 ) Total E = m(100 m 2 /s 2 )

We can leave the mass value as it appears for now. As we complete each part, you will see what happens to this variable.

The cart is at ground level at point B, so h = 0 m.

Total E = ½mv 2  + mgh Total E = ½mv 2  + mg(0 m) Total E = ½mv 2

All of the energy at this point is kinetic energy. Since total energy is conserved, the total energy at point B is the same as the total energy at point A.

Total E at A = Total Energy at B m(100 m 2 /s 2 ) = ½mv 2

Divide both sides by m 100 m 2 /s 2 = ½v 2

Multiply both sides by 2 200 m 2 /s 2 = v 2

v = 14.1 m/s

The velocity at point B is 14.1 m/s.

At point C, we know only a value for h (h = 3 m).

Total E = ½mv 2 + mgh Total E = ½mv 2 + mg(3 m)

As before, the total energy is conserved. Total energy at A = total energy at C.

m(100 m 2 /s 2 ) = ½mv 2 + m(9.8 m/s 2 )(3 m) m(100 m 2 /s 2 ) = ½mv 2 + m(29.4 m 2 /s 2 )

Divide both sides by m

100 m 2 /s 2 = ½v 2 + 29.4 m 2 /s 2 ½v 2 = (100 – 29.4) m 2 /s 2 ½v 2 = 70.6 m 2 /s 2 v 2 = 141.2 m 2 /s 2 v = 11.9 m/s

The velocity at point C is 11.9 m/s.

The cart will reach its maximum height when the cart stops or v = 0 m/s.

Total E = ½mv 2 + mgh Total E = ½m(0 m/s) 2 + mgh Total E = mgh

Since total energy is conserved, the total energy at point A is the same as the total energy at point D.

m(100 m 2 /s 2 ) = mgh

100 m 2 /s 2  = gh

100 m 2 /s 2  = (9.8 m/s 2 ) h

The maximum height of the cart is 10.2 m.

A) The velocity of the cart at ground level is 14.1 m/s. B) The velocity of the cart at a height of 3 m is 11.9 m/s. C) The maximum height of the cart is 10.2 m.

This type of problem has one main key point: total energy is conserved at all points of the system. If you know the total energy at one point, you know the total energy at all points.

Related Posts

Potential and Kinetic Energy

Energy is the capacity to do work .

The unit of energy is J (Joule) which is also kg m 2 /s 2 (kilogram meter squared per second squared)

Energy can be in many forms! Here we look at Potential Energy (PE) and Kinetic Energy (KE).

Potential Energy and Kinetic Energy

hammer

  • when raised up has potential energy (the energy of position or state)
  • when falling down has kinetic energy (the energy of motion)

Potential energy (PE) is stored energy due to position or state

bow and arrow

  • a raised hammer has PE due to gravity.
  • fuel and explosives have Chemical PE
  • a coiled spring or a drawn bow also have PE due to their state

Kinetic energy (KE) is energy of motion

car moving

From PE to KE

skydivers

For a good example of PE and KE have a play with a pendulum .

Gravitational Potential Energy

When the PE is due to an objects height then:

PE due to gravity = m g h

  • m is the objects mass (kg)
  • g is the "gravitational field strength" of 9.8 m/s 2 near the Earth's surface
  • h is height (m)

Example: This 2 kg hammer is 0.4 m up. What is it's PE?

Kinetic energy.

The formula is:

KE = ½ m v 2

  • m is the object's mass (kg)
  • v is the object's speed (m/s)

Example: What is the KE of a 1500 kg car going at suburban speed of 14 m/s (about 50 km/h or 30 mph)?

KE = ½ × 1500 kg × (14 m/s) 2

KE = 147,000 kg m 2 /s 2

KE = 147 kJ

Let's double the speed!

Example: The same car is now going at highway speed of 28 m/s (about 100 km/h or 60 mph)?

KE = ½ × 1500 kg × (28 m/s) 2

KE = 588,000 kg m 2 /s 2

KE = 588 kJ

Wow! that is a big increase in energy! Highway speed is way more dangerous.

Double the speed and the KE increases by four times. Very important to know

moon

A 1 kg meteorite strikes the Moon at 11 km/s. How much KE is that?

KE = ½ × 1 kg × (11,000 m/s) 2

KE = 60,500,000 J

KE = 60.5 MJ

That is 100 times the energy of a car going at highway speed.

When falling, an object's PE due to gravity converts into KE and also heat due to air resistance.

Let's drop something!

Example: We drop this 0.1 kg apple 1 m. What speed does it hit the ground with?

At 1 m above the ground it's Potential Energy is

PE = 0.1 kg × 9.8 m/s 2 × 1 m

PE = 0.98 kg m 2 /s 2

Ignoring air resistance (which is small for this little drop anyway) that PE gets converted into KE:

Swap sides and rearrange:

½ m v 2 = KE

v 2 = 2 × KE / m

v = √( 2 × KE / m )

Now put PE into KE and we get:

v = √( 2 × 0.98 kg m 2 /s 2 / 0.1 kg )

v = √( 19.6 m 2 /s 2 )

v = 4.427... m/s

Note: for velocity we can combine the formulas like this:

Velocity from KE:   v = √( 2 × KE / m )
Put in formula for PE:   v = √( 2 × mgh / m )
Cancel m/m:   v = √( 2gh )

The mass does not matter! It is all about height and gravity. For our earlier example:

v = √( 2gh )

v = √( 2 × 9.8 m/s 2 × 1 m )

  • Energy is the ability to do work

StickMan Physics

StickMan Physics

Animated Physics Lessons

Mechanical Energy Problem Solutions

Mechanical energy problems and solutions.

See examples of mechanical energy problems involving kinetic energy, potential energy, and the conservation of energy. Check your work with ours.

1. How much gravitational potential energy do you have when you lift a 15 N object 10 meters off the ground?

ME Problem 1

2. How much gravitational potential energy is in a 20 kg mass when 0.6 meters above the ground?

ME Problem 2

3. How much gravitational potential energy does a 35 kg boulder have when 30 meters off the ground?

ME Problem 3

4. How many times greater is an objects potential energy when three times higher?

If you need help on ratio problems click the link below:

Rule of Ones: analyzing equations to determine how other variables change

Example 4 Mechanical Energy Solution

5.  How much kinetic energy does a 0.15 kg ball thrown at 24 m/s have?

ME Problem 5

6.  How many times greater is the kinetic energy of a ball that is going five times faster?

ME Problem 6

7.  How much kinetic energy does a 1.2 kg ball have the moment it hits the ground 3.5 meters below when it starts from rest?

I cancelled out the initial kinetic energy because:

  • KE i = ½ mv f 2
  • KE i = (½)(3.5)(0 2 ) = 0 J

I cancelled out the final potential energy because:

  • PE f = mgh f
  • PE f = (3.5)(9.8)(0) = 0 J

ME Problem 7 Solution

8.  How fast is a 1.2 kg ball traveling the moment it hits the ground 3.5 meters below when it starts from rest?

(Note: In many of these problems I could cancel out mass but did not since it was provided)

Since I did not cancel out mass I could answer the following questions if asked:

  • How much mechanical energy did you have at the beginning? (41.6 J)
  • How much kinetic energy did you have at the beginning? (0 J)
  • How much potential energy did you have at the beginning? (41.6 J)
  • How much potential energy do you have at the end? (0 J)

If I cancelled out mass in my work it would not show the actual initial potential energy since PE i = mgh and not just gh.

ME Problem 8 Solution

9.  A 3.5 kg ball fell from a height of 12 meters.  How fast is it traveling when its still 5 meters off the ground?

ME Problem 9

10. An 85kg roller coaster cart is traveling 4 m/s at the top of a hill 50 meters off the ground.  How fast is it traveling at top of a second hill 20 meters off the ground?

ME Problem 10

  • Back to the Mechanical Energy Page
  • Back to the Main Work, Power, Mechanical Energy, and Simple Machines Page
  • Back to the  Stickman Physics Home Page
  • For video tutorials and other physics resources check out  HoldensClass.com
  • Find many of your animation resources in one place at the  StickMan Physics Gallery
  • Equation Sheet

Terms and Conditions - Privacy Policy

FREE K-12 standards-aligned STEM

curriculum for educators everywhere!

Find more at TeachEngineering.org .

  • TeachEngineering
  • Kinetic and Potential Energy of Motion

Lesson Kinetic and Potential Energy of Motion

Grade Level: 8 (7-9)

Time Required: 45 minutes

Lesson Dependency: None

Subject Areas: Physical Science, Physics

NGSS Performance Expectations:

NGSS Three Dimensional Triangle

Energize your students with the resources featured here, by grade band, to help them make sense of real-world phenomena related to energy!

  • Print lesson and its associated curriculum

Curriculum in this Unit Units serve as guides to a particular content or subject area. Nested under units are lessons (in purple) and hands-on activities (in blue). Note that not all lessons and activities will exist under a unit, and instead may exist as "standalone" curriculum.

  • Swinging Pendulum
  • Swinging Pendulum (for High School)
  • Bouncing Balls: Collisions, Momentum & Math in Sports
  • Bouncing Balls: Collisions, Momentum & Math (for High School)
  • Energy in Collisions: Rolling Ramp and Review
  • Energy in Collisions: Rolling Ramp and Review (for High School)
Unit Lesson Activity

TE Newsletter

Engineering connection, learning objectives, more curriculum like this, introduction/motivation, associated activities, lesson closure, vocabulary/definitions, user comments & tips.

Engineers make a world of difference

Mechanical engineers are concerned about the mechanics of energy — how it is generated, stored and moved. Product design engineers apply the principles of potential and kinetic energy when they design consumer products. For example, a pencil sharpener employs mechanical energy and electrical energy. When designing a roller coaster, mechanical and civil engineers ensure that there is sufficient potential energy (which is converted to kinetic energy) to move the cars through the entire roller coaster ride.

After this lesson, students should be able to:

  • Recognize that engineers need to understand the many different forms of energy in order to design useful products.
  • Explain the concepts of kinetic and potential energy.
  • Understand that energy can change from one form into another.
  • Understand that energy can be described by equations.

Educational Standards Each TeachEngineering lesson or activity is correlated to one or more K-12 science, technology, engineering or math (STEM) educational standards. All 100,000+ K-12 STEM standards covered in TeachEngineering are collected, maintained and packaged by the Achievement Standards Network (ASN) , a project of D2L (www.achievementstandards.org). In the ASN, standards are hierarchically structured: first by source; e.g. , by state; within source by type; e.g. , science or mathematics; within type by subtype, then by grade, etc .

Ngss: next generation science standards - science.

NGSS Performance Expectation

MS-PS3-5. Construct, use, and present arguments to support the claim that when the kinetic energy of an object changes, energy is transferred to or from the object. (Grades 6 - 8)

Do you agree with this alignment? Thanks for your feedback!

This lesson focuses on the following aspects of NGSS:
Science & Engineering Practices Disciplinary Core Ideas Crosscutting Concepts
Science knowledge is based upon logical and conceptual connections between evidence and explanations.

Alignment agreement: Thanks for your feedback!

When the motion energy of an object changes, there is inevitably some other change in energy at the same time.

Alignment agreement: Thanks for your feedback!

Energy may take different forms (e.g. energy in fields, thermal energy, energy of motion).

Alignment agreement: Thanks for your feedback!

Common Core State Standards - Math

View aligned curriculum

Do you agree with this alignment? Thanks for your feedback!

International Technology and Engineering Educators Association - Technology

State standards, colorado - math, colorado - science.

Begin by showing the class three items: 1) an item of food (such as a bagel, banana or can of soda water), 2) a battery, and 3) you, standing on a stool or chair. Ask the class what these three things have in common. The answer is energy. The food contains chemical energy that is used by the body as fuel. The battery contains electrical energy (in the form of electrical, potential or stored energy), which can be used by a flashlight or a portable CD player. A person standing on a stool has potential energy (sometimes called gravitational potential energy) that could be used to crush a can, smash the banana, or really hurt the foot of someone standing under you. Do a dramatic demonstration of jumping down on the banana or an empty soda can. (Be careful! Banana peels are slippery!) Explain the ideas of potential energy and kinetic energy as two different kinds of mechanical energy . Give definitions of each and present the equations, carefully explaining each variable, as discussed in the next section,

PE = mass x g x height

KE = 1/2 m x v 2

An image of a full roller coaster going around a loop.

Lesson Background and Concepts for Teachers

Whenever something moves, you can see the change in energy of that system. Energy can make things move or cause a change in the position or state of an object. Energy can be defined as the capacity for doing work. Work is done when a force moves an object over a given distance. The capacity for work, or energy, can come in many different forms. Examples of such forms are mechanical, electrical, chemical or nuclear energy.

This lesson introduces mechanical energy , the form of energy that is easiest to observe on a daily basis. All moving objects have mechanical energy. There are two types of mechanical energy: potential energy and kinetic energy. Potential energy is the energy that an object has because of its position and is measured in Joules (J). Potential energy can also be thought of as stored energy. Kinetic energy is the energy an object has because of its motion and is also measured in Joules (J). Due to the principle of conservation of energy, energy can change its form (potential, kinetic, heat/thermal, electrical, light, sound, etc.) but it is never created or destroyed.

Within the context of mechanical energy, potential energy is a result of an object's position, mass and the acceleration of gravity. A book resting on the edge of a table has potential energy; if you were to nudge it off the edge, the book would fall. It is sometimes called gravitational potential energy ( PE ). It can be expressed mathematically as follows:

PE = mass x g x height or PE = weight x height

where PE is the potential energy, and g is the acceleration due to gravity. At sea level, g = 9.81 meters/sec 2 or 32.2 feet/sec 2 . In the metric system, we would commonly use mass in kilograms or grams with the first equation. With English units it is common to use weight in pounds with the second equation.

Kinetic energy ( KE ) is energy of motion. Any object that is moving has kinetic energy. An example is a baseball that has been thrown. The kinetic energy depends on both mass and velocity and can be expressed mathematically as follows:

Here KE stands for kinetic energy. Note that a change in the velocity will have a much greater effect on the amount of kinetic energy because that term is squared. The total amount of mechanical energy in a system is the sum of both potential and kinetic energy, also measured in Joules (J).

Total Mechanical Energy = Potential Energy + Kinetic Energy

Engineers must understand both potential and kinetic energy. A simple example would be the design of a roller coaster — a project that involves both mechanical and civil engineers. At the beginning of the roller coaster, the cars must have enough potential energy to power them for the rest of the ride. This can be done by raising the cars to a great height. Then, the increased potential energy of the cars is converted into enough kinetic energy to keep them in motion for the length of the track. This is why roller coaters usually start with a big hill. As the cars start down the first hill, potential energy is changed into kinetic energy and the cars pick up speed. Engineers design the roller coaster to have enough energy to complete the course and to overcome the energy-draining effect of friction.

Watch this activity on YouTube

Restate that both potential energy and kinetic energy are forms of mechanical energy. Potential energy is the energy of position and kinetic energy is the energy of motion. A ball that you hold in your hand has potential energy, while a ball that you throw has kinetic energy. These two forms of energy can be transformed back and forth. When you drop a ball, you demonstrate an example of potential energy changing into kinetic energy.

Explain that energy is an important engineering concept. Engineers need to understand the many different forms of energy so that they can design useful products. An electric pencil sharpener serves to illustrate the point. First, the designer needs to know the amount of kinetic energy the spinning blades need in order to successfully shave off the end of the pencil. Then, the designer must choose an appropriately-powered motor to supply the necessary energy. Finally, the designer must know the electrical energy requirements of the motor in order for the motor to properly do its assigned task.

conservation of energy: A principle stating that the total energy of an isolated system remains constant regardless of changes within the system. Energy can neither be created nor destroyed.

energy: Energy is the capacity to do work.

kinetic energy: The energy of motion.

mechanical energy: Energy that is composed of both potential energy and kinetic energy.

potential energy: The energy of position, or stored energy.

Pre-Lesson Assessment

Discussion Questions: Solicit, integrate and summarize student responses.

  • What are examples of dangerous unsafe placement of objects? (Possible answers: Boulders on the edge of a cliff, dishes barely on shelves, etc.).

Post-Introduction Assessment

Question/Answer: Ask the students and discuss as a class:

  • What has more potential energy: a boulder on the ground or a feather 10 feet in the air? (Answer: The feather because the boulder is on the ground and has zero potential energy. However, if the boulder was 1 mm off the ground, it would probably have more potential energy.)

Lesson Summary Assessment

Group Brainstorm: Give groups of students each a ball (example, tennis ball). Remind them that energy can be converted from potential to kinetic and vice versa. Write a question on the board and have them brainstorm the answer in their groups. Have the students record their answers in their journals or on a sheet of paper and hand it in. Discuss the student groups' answers with the class.

  • How can you throw a ball and have its energy change from kinetic to potential and back to kinetic without touching the ball once it relases from your hand? (Answer: Throw it straight up in the air.)

Calculating: Have students practice problems solving for potential energy and kinetic energy:

  • If a mass that weighs 8 kg is held at a height of 10 m, what is its potential energy? (Answer: PE = (8 kg)*(9.8 m/s 2 )*(10 m) = 784 kg*m 2 /s 2 = 784 J)
  • Now consider an object with a kinetic energy of 800 J and a mass of 12 kg. What is its velocity? (Answer: v = sqrt(2*KE/m) = sqrt((2 * 800 J)/12 kg) = 11.55 m/s)

Lesson Extension Activities

There is another form of potential energy, not related to height, which is called spring potential or elastic potential energy . In this case, energy is stored when you compress or elongate a spring. Have the students search the Internet or library for the equation of spring potential energy and explain what the variables in the equation represent. The answer is

PE spring = ½ k∙x 2

where k is the spring constant measured in N/m (Newton/meters) and x is how far the spring is compressed or stretched measured in m (meters).

sample problem solving in potential and kinetic energy

This activity shows students the engineering importance of understanding the laws of mechanical energy. More specifically, it demonstrates how potential energy can be converted to kinetic energy and back again. Given a pendulum height, students calculate and predict how fast the pendulum will swing ...

preview of 'Swinging Pendulum (for High School)' Activity

This activity demonstrates how potential energy (PE) can be converted to kinetic energy (KE) and back again. Given a pendulum height, students calculate and predict how fast the pendulum will swing by understanding conservation of energy and using the equations for PE and KE.

preview of 'Swinging Pendulum' Activity

High school students learn how engineers mathematically design roller coaster paths using the approach that a curved path can be approximated by a sequence of many short inclines. They apply basic calculus and the work-energy theorem for non-conservative forces to quantify the friction along a curve...

preview of 'A Tale of Friction ' Lesson

Students explore the physics exploited by engineers in designing today's roller coasters, including potential and kinetic energy, friction and gravity. During the associated activity, students design, build and analyze model roller coasters they make using foam tubing and marbles (as the cars).

preview of 'Physics of Roller Coasters' Lesson

Argonne Transportation - Laser Glazing of Rails. September 29, 2003. Argonne National Laboratory, Transportation Technology R&D Center. October 15, 2003. http://www.anl.gov/index.html

Asimov, Isaac. The History of Physics. New York: Walker & Co., 1984.

Jones, Edwin R. and Richard L. Childers. Contemporary College Physics. Reading, MA: Addison-Wesley Publishing Co., 1993.

Kahan, Peter. Science Explorer: Motion, Forces, and Energy. Upper Saddle River, NJ: Prentice Hall, 2000.

Luehmann, April. Give Me Energy. June 12, 2003. Science and Mathematics Initiative for Learning Enhancement, Illinois Institute of Technology. October 15, 2003. http://www.iit.edu/~smile/ph9407.html

Nave, C.R. HyperPhysics. 2000. Department of Physics and Astronomy, Georgia State University. October 15, 2003. hyperphysics.phy-astr.gsu.edu/hbase/hframe.html

The Atoms Family - The Mummy's Tomb – Raceways. Miami Museum of Science and Space Transit Planetarium. October 15, 2003. http://www.miamisci.org/af/sln/mummy/raceways.html

Other Related Information

Browse the NGSS Engineering-aligned Physics Curriculum hub for additional Physics and Physical Science curriculum featuring Engineering.

Contributors

Supporting program, acknowledgements.

The contents of this digital library curriculum were developed under a grant from the Fund for the Improvement of Postsecondary Education (FIPSE), U.S. Department of Education and National Science Foundation GK-12 grant no. 0338326. However, these contents do not necessarily represent the policies of the Department of Education or National Science Foundation, and you should not assume endorsement by the federal government.

Last modified: December 14, 2022

Youtube

  • TPC and eLearning
  • What's NEW at TPC?
  • Read Watch Interact
  • Practice Review Test
  • Teacher-Tools
  • Subscription Selection
  • Seat Calculator
  • Ad Free Account
  • Edit Profile Settings
  • Classes (Version 2)
  • Student Progress Edit
  • Task Properties
  • Export Student Progress
  • Task, Activities, and Scores
  • Metric Conversions Questions
  • Metric System Questions
  • Metric Estimation Questions
  • Significant Digits Questions
  • Proportional Reasoning
  • Acceleration
  • Distance-Displacement
  • Dots and Graphs
  • Graph That Motion
  • Match That Graph
  • Name That Motion
  • Motion Diagrams
  • Pos'n Time Graphs Numerical
  • Pos'n Time Graphs Conceptual
  • Up And Down - Questions
  • Balanced vs. Unbalanced Forces
  • Change of State
  • Force and Motion
  • Mass and Weight
  • Match That Free-Body Diagram
  • Net Force (and Acceleration) Ranking Tasks
  • Newton's Second Law
  • Normal Force Card Sort
  • Recognizing Forces
  • Air Resistance and Skydiving
  • Solve It! with Newton's Second Law
  • Which One Doesn't Belong?
  • Component Addition Questions
  • Head-to-Tail Vector Addition
  • Projectile Mathematics
  • Trajectory - Angle Launched Projectiles
  • Trajectory - Horizontally Launched Projectiles
  • Vector Addition
  • Vector Direction
  • Which One Doesn't Belong? Projectile Motion
  • Forces in 2-Dimensions
  • Being Impulsive About Momentum
  • Explosions - Law Breakers
  • Hit and Stick Collisions - Law Breakers
  • Case Studies: Impulse and Force
  • Impulse-Momentum Change Table
  • Keeping Track of Momentum - Hit and Stick
  • Keeping Track of Momentum - Hit and Bounce
  • What's Up (and Down) with KE and PE?
  • Energy Conservation Questions
  • Energy Dissipation Questions
  • Energy Ranking Tasks
  • LOL Charts (a.k.a., Energy Bar Charts)
  • Match That Bar Chart
  • Words and Charts Questions
  • Name That Energy
  • Stepping Up with PE and KE Questions
  • Case Studies - Circular Motion
  • Circular Logic
  • Forces and Free-Body Diagrams in Circular Motion
  • Gravitational Field Strength
  • Universal Gravitation
  • Angular Position and Displacement
  • Linear and Angular Velocity
  • Angular Acceleration
  • Rotational Inertia
  • Balanced vs. Unbalanced Torques
  • Getting a Handle on Torque
  • Torque-ing About Rotation
  • Properties of Matter
  • Fluid Pressure
  • Buoyant Force
  • Sinking, Floating, and Hanging
  • Pascal's Principle
  • Flow Velocity
  • Bernoulli's Principle
  • Balloon Interactions
  • Charge and Charging
  • Charge Interactions
  • Charging by Induction
  • Conductors and Insulators
  • Coulombs Law
  • Electric Field
  • Electric Field Intensity
  • Polarization
  • Case Studies: Electric Power
  • Know Your Potential
  • Light Bulb Anatomy
  • I = ∆V/R Equations as a Guide to Thinking
  • Parallel Circuits - ∆V = I•R Calculations
  • Resistance Ranking Tasks
  • Series Circuits - ∆V = I•R Calculations
  • Series vs. Parallel Circuits
  • Equivalent Resistance
  • Period and Frequency of a Pendulum
  • Pendulum Motion: Velocity and Force
  • Energy of a Pendulum
  • Period and Frequency of a Mass on a Spring
  • Horizontal Springs: Velocity and Force
  • Vertical Springs: Velocity and Force
  • Energy of a Mass on a Spring
  • Decibel Scale
  • Frequency and Period
  • Closed-End Air Columns
  • Name That Harmonic: Strings
  • Rocking the Boat
  • Wave Basics
  • Matching Pairs: Wave Characteristics
  • Wave Interference
  • Waves - Case Studies
  • Color Addition and Subtraction
  • Color Filters
  • If This, Then That: Color Subtraction
  • Light Intensity
  • Color Pigments
  • Converging Lenses
  • Curved Mirror Images
  • Law of Reflection
  • Refraction and Lenses
  • Total Internal Reflection
  • Who Can See Who?
  • Formulas and Atom Counting
  • Atomic Models
  • Bond Polarity
  • Entropy Questions
  • Cell Voltage Questions
  • Heat of Formation Questions
  • Reduction Potential Questions
  • Oxidation States Questions
  • Measuring the Quantity of Heat
  • Hess's Law
  • Oxidation-Reduction Questions
  • Galvanic Cells Questions
  • Thermal Stoichiometry
  • Molecular Polarity
  • Quantum Mechanics
  • Balancing Chemical Equations
  • Bronsted-Lowry Model of Acids and Bases
  • Classification of Matter
  • Collision Model of Reaction Rates
  • Density Ranking Tasks
  • Dissociation Reactions
  • Complete Electron Configurations
  • Elemental Measures
  • Enthalpy Change Questions
  • Equilibrium Concept
  • Equilibrium Constant Expression
  • Equilibrium Calculations - Questions
  • Equilibrium ICE Table
  • Intermolecular Forces Questions
  • Ionic Bonding
  • Lewis Electron Dot Structures
  • Limiting Reactants
  • Line Spectra Questions
  • Mass Stoichiometry
  • Measurement and Numbers
  • Metals, Nonmetals, and Metalloids
  • Metric Estimations
  • Metric System
  • Molarity Ranking Tasks
  • Mole Conversions
  • Name That Element
  • Names to Formulas
  • Names to Formulas 2
  • Nuclear Decay
  • Particles, Words, and Formulas
  • Periodic Trends
  • Precipitation Reactions and Net Ionic Equations
  • Pressure Concepts
  • Pressure-Temperature Gas Law
  • Pressure-Volume Gas Law
  • Chemical Reaction Types
  • Significant Digits and Measurement
  • States Of Matter Exercise
  • Stoichiometry Law Breakers
  • Stoichiometry - Math Relationships
  • Subatomic Particles
  • Spontaneity and Driving Forces
  • Gibbs Free Energy
  • Volume-Temperature Gas Law
  • Acid-Base Properties
  • Energy and Chemical Reactions
  • Chemical and Physical Properties
  • Valence Shell Electron Pair Repulsion Theory
  • Writing Balanced Chemical Equations
  • Mission CG1
  • Mission CG10
  • Mission CG2
  • Mission CG3
  • Mission CG4
  • Mission CG5
  • Mission CG6
  • Mission CG7
  • Mission CG8
  • Mission CG9
  • Mission EC1
  • Mission EC10
  • Mission EC11
  • Mission EC12
  • Mission EC2
  • Mission EC3
  • Mission EC4
  • Mission EC5
  • Mission EC6
  • Mission EC7
  • Mission EC8
  • Mission EC9
  • Mission RL1
  • Mission RL2
  • Mission RL3
  • Mission RL4
  • Mission RL5
  • Mission RL6
  • Mission KG7
  • Mission RL8
  • Mission KG9
  • Mission RL10
  • Mission RL11
  • Mission RM1
  • Mission RM2
  • Mission RM3
  • Mission RM4
  • Mission RM5
  • Mission RM6
  • Mission RM8
  • Mission RM10
  • Mission LC1
  • Mission RM11
  • Mission LC2
  • Mission LC3
  • Mission LC4
  • Mission LC5
  • Mission LC6
  • Mission LC8
  • Mission SM1
  • Mission SM2
  • Mission SM3
  • Mission SM4
  • Mission SM5
  • Mission SM6
  • Mission SM8
  • Mission SM10
  • Mission KG10
  • Mission SM11
  • Mission KG2
  • Mission KG3
  • Mission KG4
  • Mission KG5
  • Mission KG6
  • Mission KG8
  • Mission KG11
  • Mission F2D1
  • Mission F2D2
  • Mission F2D3
  • Mission F2D4
  • Mission F2D5
  • Mission F2D6
  • Mission KC1
  • Mission KC2
  • Mission KC3
  • Mission KC4
  • Mission KC5
  • Mission KC6
  • Mission KC7
  • Mission KC8
  • Mission AAA
  • Mission SM9
  • Mission LC7
  • Mission LC9
  • Mission NL1
  • Mission NL2
  • Mission NL3
  • Mission NL4
  • Mission NL5
  • Mission NL6
  • Mission NL7
  • Mission NL8
  • Mission NL9
  • Mission NL10
  • Mission NL11
  • Mission NL12
  • Mission MC1
  • Mission MC10
  • Mission MC2
  • Mission MC3
  • Mission MC4
  • Mission MC5
  • Mission MC6
  • Mission MC7
  • Mission MC8
  • Mission MC9
  • Mission RM7
  • Mission RM9
  • Mission RL7
  • Mission RL9
  • Mission SM7
  • Mission SE1
  • Mission SE10
  • Mission SE11
  • Mission SE12
  • Mission SE2
  • Mission SE3
  • Mission SE4
  • Mission SE5
  • Mission SE6
  • Mission SE7
  • Mission SE8
  • Mission SE9
  • Mission VP1
  • Mission VP10
  • Mission VP2
  • Mission VP3
  • Mission VP4
  • Mission VP5
  • Mission VP6
  • Mission VP7
  • Mission VP8
  • Mission VP9
  • Mission WM1
  • Mission WM2
  • Mission WM3
  • Mission WM4
  • Mission WM5
  • Mission WM6
  • Mission WM7
  • Mission WM8
  • Mission WE1
  • Mission WE10
  • Mission WE2
  • Mission WE3
  • Mission WE4
  • Mission WE5
  • Mission WE6
  • Mission WE7
  • Mission WE8
  • Mission WE9
  • Vector Walk Interactive
  • Name That Motion Interactive
  • Kinematic Graphing 1 Concept Checker
  • Kinematic Graphing 2 Concept Checker
  • Graph That Motion Interactive
  • Two Stage Rocket Interactive
  • Rocket Sled Concept Checker
  • Force Concept Checker
  • Free-Body Diagrams Concept Checker
  • Free-Body Diagrams The Sequel Concept Checker
  • Skydiving Concept Checker
  • Elevator Ride Concept Checker
  • Vector Addition Concept Checker
  • Vector Walk in Two Dimensions Interactive
  • Name That Vector Interactive
  • River Boat Simulator Concept Checker
  • Projectile Simulator 2 Concept Checker
  • Projectile Simulator 3 Concept Checker
  • Hit the Target Interactive
  • Turd the Target 1 Interactive
  • Turd the Target 2 Interactive
  • Balance It Interactive
  • Go For The Gold Interactive
  • Egg Drop Concept Checker
  • Fish Catch Concept Checker
  • Exploding Carts Concept Checker
  • Collision Carts - Inelastic Collisions Concept Checker
  • Its All Uphill Concept Checker
  • Stopping Distance Concept Checker
  • Chart That Motion Interactive
  • Roller Coaster Model Concept Checker
  • Uniform Circular Motion Concept Checker
  • Horizontal Circle Simulation Concept Checker
  • Vertical Circle Simulation Concept Checker
  • Race Track Concept Checker
  • Gravitational Fields Concept Checker
  • Orbital Motion Concept Checker
  • Angular Acceleration Concept Checker
  • Balance Beam Concept Checker
  • Torque Balancer Concept Checker
  • Aluminum Can Polarization Concept Checker
  • Charging Concept Checker
  • Name That Charge Simulation
  • Coulomb's Law Concept Checker
  • Electric Field Lines Concept Checker
  • Put the Charge in the Goal Concept Checker
  • Circuit Builder Concept Checker (Series Circuits)
  • Circuit Builder Concept Checker (Parallel Circuits)
  • Circuit Builder Concept Checker (∆V-I-R)
  • Circuit Builder Concept Checker (Voltage Drop)
  • Equivalent Resistance Interactive
  • Pendulum Motion Simulation Concept Checker
  • Mass on a Spring Simulation Concept Checker
  • Particle Wave Simulation Concept Checker
  • Boundary Behavior Simulation Concept Checker
  • Slinky Wave Simulator Concept Checker
  • Simple Wave Simulator Concept Checker
  • Wave Addition Simulation Concept Checker
  • Standing Wave Maker Simulation Concept Checker
  • Color Addition Concept Checker
  • Painting With CMY Concept Checker
  • Stage Lighting Concept Checker
  • Filtering Away Concept Checker
  • InterferencePatterns Concept Checker
  • Young's Experiment Interactive
  • Plane Mirror Images Interactive
  • Who Can See Who Concept Checker
  • Optics Bench (Mirrors) Concept Checker
  • Name That Image (Mirrors) Interactive
  • Refraction Concept Checker
  • Total Internal Reflection Concept Checker
  • Optics Bench (Lenses) Concept Checker
  • Kinematics Preview
  • Velocity Time Graphs Preview
  • Moving Cart on an Inclined Plane Preview
  • Stopping Distance Preview
  • Cart, Bricks, and Bands Preview
  • Fan Cart Study Preview
  • Friction Preview
  • Coffee Filter Lab Preview
  • Friction, Speed, and Stopping Distance Preview
  • Up and Down Preview
  • Projectile Range Preview
  • Ballistics Preview
  • Juggling Preview
  • Marshmallow Launcher Preview
  • Air Bag Safety Preview
  • Colliding Carts Preview
  • Collisions Preview
  • Engineering Safer Helmets Preview
  • Push the Plow Preview
  • Its All Uphill Preview
  • Energy on an Incline Preview
  • Modeling Roller Coasters Preview
  • Hot Wheels Stopping Distance Preview
  • Ball Bat Collision Preview
  • Energy in Fields Preview
  • Weightlessness Training Preview
  • Roller Coaster Loops Preview
  • Universal Gravitation Preview
  • Keplers Laws Preview
  • Kepler's Third Law Preview
  • Charge Interactions Preview
  • Sticky Tape Experiments Preview
  • Wire Gauge Preview
  • Voltage, Current, and Resistance Preview
  • Light Bulb Resistance Preview
  • Series and Parallel Circuits Preview
  • Thermal Equilibrium Preview
  • Linear Expansion Preview
  • Heating Curves Preview
  • Electricity and Magnetism - Part 1 Preview
  • Electricity and Magnetism - Part 2 Preview
  • Vibrating Mass on a Spring Preview
  • Period of a Pendulum Preview
  • Wave Speed Preview
  • Slinky-Experiments Preview
  • Standing Waves in a Rope Preview
  • Sound as a Pressure Wave Preview
  • DeciBel Scale Preview
  • DeciBels, Phons, and Sones Preview
  • Sound of Music Preview
  • Shedding Light on Light Bulbs Preview
  • Models of Light Preview
  • Electromagnetic Radiation Preview
  • Electromagnetic Spectrum Preview
  • EM Wave Communication Preview
  • Digitized Data Preview
  • Light Intensity Preview
  • Concave Mirrors Preview
  • Object Image Relations Preview
  • Snells Law Preview
  • Reflection vs. Transmission Preview
  • Magnification Lab Preview
  • Reactivity Preview
  • Ions and the Periodic Table Preview
  • Periodic Trends Preview
  • Chemical Reactions Preview
  • Intermolecular Forces Preview
  • Melting Points and Boiling Points Preview
  • Bond Energy and Reactions Preview
  • Reaction Rates Preview
  • Ammonia Factory Preview
  • Stoichiometry Preview
  • Nuclear Chemistry Preview
  • Gaining Teacher Access
  • Algebra Based Physics Course
  • Teacher Notes
  • Tasks and Classes
  • Tasks - Classic
  • Subscription
  • Subscription Locator
  • 1-D Kinematics
  • Newton's Laws
  • Vectors - Motion and Forces in Two Dimensions
  • Momentum and Its Conservation
  • Work and Energy
  • Circular Motion and Satellite Motion
  • Thermal Physics
  • Static Electricity
  • Electric Circuits
  • Vibrations and Waves
  • Sound Waves and Music
  • Light and Color
  • Reflection and Mirrors
  • About the Physics Interactives
  • Task Tracker
  • Usage Policy
  • Newtons Laws
  • Vectors and Projectiles
  • Forces in 2D
  • Momentum and Collisions
  • Circular and Satellite Motion
  • Balance and Rotation
  • Electromagnetism
  • Waves and Sound
  • Atomic Physics
  • Forces in Two Dimensions
  • Work, Energy, and Power
  • Circular Motion and Gravitation
  • Sound Waves
  • 1-Dimensional Kinematics
  • Circular, Satellite, and Rotational Motion
  • Einstein's Theory of Special Relativity
  • Waves, Sound and Light
  • QuickTime Movies
  • About the Concept Builders
  • Pricing For Schools
  • Directions for Version 2
  • Measurement and Units
  • Relationships and Graphs
  • Rotation and Balance
  • Vibrational Motion
  • Reflection and Refraction
  • Teacher Accounts
  • Task Tracker Directions
  • Kinematic Concepts
  • Kinematic Graphing
  • Wave Motion
  • Sound and Music
  • About CalcPad
  • 1D Kinematics
  • Vectors and Forces in 2D
  • Simple Harmonic Motion
  • Rotational Kinematics
  • Rotation and Torque
  • Rotational Dynamics
  • Electric Fields, Potential, and Capacitance
  • Transient RC Circuits
  • Light Waves
  • Units and Measurement
  • Stoichiometry
  • Molarity and Solutions
  • Thermal Chemistry
  • Acids and Bases
  • Kinetics and Equilibrium
  • Solution Equilibria
  • Oxidation-Reduction
  • Nuclear Chemistry
  • Newton's Laws of Motion
  • Work and Energy Packet
  • Static Electricity Review
  • NGSS Alignments
  • 1D-Kinematics
  • Projectiles
  • Circular Motion
  • Magnetism and Electromagnetism
  • Graphing Practice
  • About the ACT
  • ACT Preparation
  • For Teachers
  • Other Resources
  • Solutions Guide
  • Solutions Guide Digital Download
  • Motion in One Dimension
  • Work, Energy and Power
  • Algebra Based Physics
  • Honors Physics
  • Other Tools
  • Frequently Asked Questions
  • Purchasing the Download
  • Purchasing the CD
  • Purchasing the Digital Download
  • About the NGSS Corner
  • NGSS Search
  • Force and Motion DCIs - High School
  • Energy DCIs - High School
  • Wave Applications DCIs - High School
  • Force and Motion PEs - High School
  • Energy PEs - High School
  • Wave Applications PEs - High School
  • Crosscutting Concepts
  • The Practices
  • Physics Topics
  • NGSS Corner: Activity List
  • NGSS Corner: Infographics
  • About the Toolkits
  • Position-Velocity-Acceleration
  • Position-Time Graphs
  • Velocity-Time Graphs
  • Newton's First Law
  • Newton's Second Law
  • Newton's Third Law
  • Terminal Velocity
  • Projectile Motion
  • Forces in 2 Dimensions
  • Impulse and Momentum Change
  • Momentum Conservation
  • Work-Energy Fundamentals
  • Work-Energy Relationship
  • Roller Coaster Physics
  • Satellite Motion
  • Electric Fields
  • Circuit Concepts
  • Series Circuits
  • Parallel Circuits
  • Describing-Waves
  • Wave Behavior Toolkit
  • Standing Wave Patterns
  • Resonating Air Columns
  • Wave Model of Light
  • Plane Mirrors
  • Curved Mirrors
  • Teacher Guide
  • Using Lab Notebooks
  • Current Electricity
  • Light Waves and Color
  • Reflection and Ray Model of Light
  • Refraction and Ray Model of Light
  • Classes (Legacy)
  • Teacher Resources
  • HomeSchool Course
  • Subscriptions

sample problem solving in potential and kinetic energy

  • Newton's Laws
  • Einstein's Theory of Special Relativity
  • About Concept Checkers
  • School Pricing
  • Newton's Laws of Motion
  • Newton's First Law
  • Newton's Third Law

Mechanics: Work, Energy and Power

Name:___________________________________________

Kinetic Energy Problemset

                                        

SHOW ALL WORK!

1.  What is the kinetic energy of a jogger with a mass of 65.0 kg traveling at a speed of 2.5 m/s?

2.  What is the mass of a baseball that has a kinetic energy of 100 J and is traveling at 5 m/s?

Write down what you know:

sample problem solving in potential and kinetic energy

3. What is the kinetic energy of a 0.5 kg soccer ball that is traveling at a speed of 3 m/s?

soccer ball

4  What is the kinetic energy of a 1 kg pie travelling at a speed of 4 m/s ?

pie

5.  What is the kinetic energy of the pie if it is thrown at 10 m/s?

6. A student is hit with a 1 kg pumpkin pie. The kinetic energy of the pie 32 J. What was the speed of the pie?

GPE = mgh | g = 9.8 m/s 2

1.  Find the gravitational potential energy of a light that has a mass of 13.0 kg and is 4.8 m above the ground.

apple tree

3.  A marble is on a table 2.4 m above the ground.  What is the mass of the marble if it has a GPE of 568 J. 

4.  A box with a mass of 12.5 kg sits on the floor.  How high would you need to lift it for it to have a GPE of 355J ?

5.  A cart at the top of a 300 m hill has a mass of  40 kg.  What is the cart’s gravitational potential energy?

6. Examine the graphic below.

What is the gravitational potential energy of the 6 kg cart as it sits the the top of the incline?  _______________

What is the KINETIC ENERGY of the cart if it is moving at a speed of 2 m/s at the bottom of the ramp? _____________

cart on ramp

Practice Problems for Kinetic and Potential Energy

Profile Picture

Students also viewed

Profile Picture

7.2 Kinetic Energy and the Work-Energy Theorem

Learning objectives.

By the end of this section, you will be able to:

  • Explain work as a transfer of energy and net work as the work done by the net force.
  • Explain and apply the work-energy theorem.

Work Transfers Energy

What happens to the work done on a system? Energy is transferred into the system, but in what form? Does it remain in the system or move on? The answers depend on the situation. For example, if the lawn mower in Figure 7.2 (a) is pushed just hard enough to keep it going at a constant speed, then energy put into the mower by the person is removed continuously by friction, and eventually leaves the system in the form of heat transfer. In contrast, work done on the briefcase by the person carrying it up stairs in Figure 7.2 (d) is stored in the briefcase-Earth system and can be recovered at any time, as shown in Figure 7.2 (e). In fact, the building of the pyramids in ancient Egypt is an example of storing energy in a system by doing work on the system. Some of the energy imparted to the stone blocks in lifting them during construction of the pyramids remains in the stone-Earth system and has the potential to do work.

In this section we begin the study of various types of work and forms of energy. We will find that some types of work leave the energy of a system constant, for example, whereas others change the system in some way, such as making it move. We will also develop definitions of important forms of energy, such as the energy of motion.

Net Work and the Work-Energy Theorem

We know from the study of Newton’s laws in Dynamics: Force and Newton's Laws of Motion that net force causes acceleration. We will see in this section that work done by the net force gives a system energy of motion, and in the process we will also find an expression for the energy of motion.

Let us start by considering the total, or net, work done on a system. Net work is defined to be the sum of work on an object. The net work can be written in terms of the net force on an object. F net F net . In equation form, this is W net = F net d cos θ W net = F net d cos θ where θ θ is the angle between the force vector and the displacement vector.

Figure 7.3 (a) shows a graph of force versus displacement for the component of the force in the direction of the displacement—that is, an F cos θ F cos θ vs. d d graph. In this case, F cos θ F cos θ is constant. You can see that the area under the graph is F d cos θ F d cos θ , or the work done. Figure 7.3 (b) shows a more general process where the force varies. The area under the curve is divided into strips, each having an average force ( F cos θ ) i ( ave ) ( F cos θ ) i ( ave ) . The work done is ( F cos θ ) i ( ave ) d i ( F cos θ ) i ( ave ) d i for each strip, and the total work done is the sum of the W i W i . Thus the total work done is the total area under the curve, a useful property to which we shall refer later.

Net work will be simpler to examine if we consider a one-dimensional situation where a force is used to accelerate an object in a direction parallel to its initial velocity. Such a situation occurs for the package on the roller belt conveyor system shown in Figure 7.4 .

The force of gravity and the normal force acting on the package are perpendicular to the displacement and do no work. Moreover, they are also equal in magnitude and opposite in direction so they cancel in calculating the net force. The net force arises solely from the horizontal applied force F app F app and the horizontal friction force f f . Thus, as expected, the net force is parallel to the displacement, so that θ = 0º θ = 0º and cos θ = 1 cos θ = 1 , and the net work is given by

The effect of the net force F net F net is to accelerate the package from v 0 v 0 to v v . The kinetic energy of the package increases, indicating that the net work done on the system is positive. (See Example 7.2 .) By using Newton’s second law, and doing some algebra, we can reach an interesting conclusion. Substituting F net = ma F net = ma from Newton’s second law gives

To get a relationship between net work and the speed given to a system by the net force acting on it, we take d = x − x 0 d = x − x 0 and use the equation studied in Motion Equations for Constant Acceleration in One Dimension for the change in speed over a distance d d if the acceleration has the constant value a a ; namely, v 2 = v 0 2 + 2 ad v 2 = v 0 2 + 2 ad (note that a a appears in the expression for the net work). Solving for acceleration gives a = v 2 − v 0 2 2 d a = v 2 − v 0 2 2 d . When a a is substituted into the preceding expression for W net W net , we obtain

The d d cancels, and we rearrange this to obtain

This expression is called the work-energy theorem , and it actually applies in general (even for forces that vary in direction and magnitude), although we have derived it for the special case of a constant force parallel to the displacement. The theorem implies that the net work on a system equals the change in the quantity 1 2 mv 2 1 2 mv 2 . This quantity is our first example of a form of energy.

The Work-Energy Theorem

The net work on a system equals the change in the quantity 1 2 mv 2 1 2 mv 2 .

The quantity 1 2 mv 2 1 2 mv 2 in the work-energy theorem is defined to be the translational kinetic energy (KE) of a mass m m moving at a speed v v . ( Translational kinetic energy is distinct from rotational kinetic energy, which is considered later.) In equation form, the translational kinetic energy,

is the energy associated with translational motion. Kinetic energy is a form of energy associated with the motion of a particle, single body, or system of objects moving together.

We are aware that it takes energy to get an object, like a car or the package in Figure 7.4 , up to speed, but it may be a bit surprising that kinetic energy is proportional to speed squared. This proportionality means, for example, that a car traveling at 100 km/h has four times the kinetic energy it has at 50 km/h, helping to explain why high-speed collisions are so devastating. We will now consider a series of examples to illustrate various aspects of work and energy.

Example 7.2

Calculating the kinetic energy of a package.

Suppose a 30.0-kg package on the roller belt conveyor system in Figure 7.4 is moving at 0.500 m/s. What is its kinetic energy?

Because the mass m m and speed v v are given, the kinetic energy can be calculated from its definition as given in the equation KE = 1 2 mv 2 KE = 1 2 mv 2 .

The kinetic energy is given by

Entering known values gives

which yields

Note that the unit of kinetic energy is the joule, the same as the unit of work, as mentioned when work was first defined. It is also interesting that, although this is a fairly massive package, its kinetic energy is not large at this relatively low speed. This fact is consistent with the observation that people can move packages like this without exhausting themselves.

Example 7.3

Determining the work to accelerate a package.

Suppose that you push on the 30.0-kg package in Figure 7.4 with a constant force of 120 N through a distance of 0.800 m, and that the opposing friction force averages 5.00 N.

(a) Calculate the net work done on the package. (b) Solve the same problem as in part (a), this time by finding the work done by each force that contributes to the net force.

Strategy and Concept for (a)

This is a motion in one dimension problem, because the downward force (from the weight of the package) and the normal force have equal magnitude and opposite direction, so that they cancel in calculating the net force, while the applied force, friction, and the displacement are all horizontal. (See Figure 7.4 .) As expected, the net work is the net force times distance.

Solution for (a)

The net force is the push force minus friction, or F net = 120 N – 5 . 00 N = 115 N F net = 120 N – 5 . 00 N = 115 N . Thus the net work is

Discussion for (a)

This value is the net work done on the package. The person actually does more work than this, because friction opposes the motion. Friction does negative work and removes some of the energy the person expends and converts it to thermal energy. The net work equals the sum of the work done by each individual force.

Strategy and Concept for (b)

The forces acting on the package are gravity, the normal force, the force of friction, and the applied force. The normal force and force of gravity are each perpendicular to the displacement, and therefore do no work.

Solution for (b)

The applied force does work.

The friction force and displacement are in opposite directions, so that θ = 180º θ = 180º , and the work done by friction is

So the amounts of work done by gravity, by the normal force, by the applied force, and by friction are, respectively,

The total work done as the sum of the work done by each force is then seen to be

Discussion for (b)

The calculated total work W total W total as the sum of the work by each force agrees, as expected, with the work W net W net done by the net force. The work done by a collection of forces acting on an object can be calculated by either approach.

Example 7.4

Determining speed from work and energy.

Find the speed of the package in Figure 7.4 at the end of the push, using work and energy concepts.

Here the work-energy theorem can be used, because we have just calculated the net work, W net W net , and the initial kinetic energy, 1 2 m v 0 2 1 2 m v 0 2 . These calculations allow us to find the final kinetic energy, 1 2 mv 2 1 2 mv 2 , and thus the final speed v v .

The work-energy theorem in equation form is

Solving for 1 2 mv 2 1 2 mv 2 gives

Solving for the final speed as requested and entering known values gives

Using work and energy, we not only arrive at an answer, we see that the final kinetic energy is the sum of the initial kinetic energy and the net work done on the package. This means that the work indeed adds to the energy of the package.

Example 7.5

Work and energy can reveal distance, too.

How far does the package in Figure 7.4 coast after the push, assuming friction remains constant? Use work and energy considerations.

We know that once the person stops pushing, friction will bring the package to rest. In terms of energy, friction does negative work until it has removed all of the package’s kinetic energy. The work done by friction is the force of friction times the distance traveled times the cosine of the angle between the friction force and displacement; hence, this gives us a way of finding the distance traveled after the person stops pushing.

The normal force and force of gravity cancel in calculating the net force. The horizontal friction force is then the net force, and it acts opposite to the displacement, so θ = 180º θ = 180º . To reduce the kinetic energy of the package to zero, the work W fr W fr by friction must be minus the kinetic energy that the package started with plus what the package accumulated due to the pushing. Thus W fr = − 95 . 75 J W fr = − 95 . 75 J . Furthermore, W fr = f d ′ cos θ = – f d ′ W fr = f d ′ cos θ = – f d ′ , where d ′ d ′ is the distance it takes to stop. Thus,

This is a reasonable distance for a package to coast on a relatively friction-free conveyor system. Note that the work done by friction is negative (the force is in the opposite direction of motion), so it removes the kinetic energy.

Some of the examples in this section can be solved without considering energy, but at the expense of missing out on gaining insights about what work and energy are doing in this situation. On the whole, solutions involving energy are generally shorter and easier than those using kinematics and dynamics alone.

As an Amazon Associate we earn from qualifying purchases.

This book may not be used in the training of large language models or otherwise be ingested into large language models or generative AI offerings without OpenStax's permission.

Want to cite, share, or modify this book? This book uses the Creative Commons Attribution License and you must attribute OpenStax.

Access for free at https://openstax.org/books/college-physics-2e/pages/1-introduction-to-science-and-the-realm-of-physics-physical-quantities-and-units
  • Authors: Paul Peter Urone, Roger Hinrichs
  • Publisher/website: OpenStax
  • Book title: College Physics 2e
  • Publication date: Jul 13, 2022
  • Location: Houston, Texas
  • Book URL: https://openstax.org/books/college-physics-2e/pages/1-introduction-to-science-and-the-realm-of-physics-physical-quantities-and-units
  • Section URL: https://openstax.org/books/college-physics-2e/pages/7-2-kinetic-energy-and-the-work-energy-theorem

© Jan 19, 2024 OpenStax. Textbook content produced by OpenStax is licensed under a Creative Commons Attribution License . The OpenStax name, OpenStax logo, OpenStax book covers, OpenStax CNX name, and OpenStax CNX logo are not subject to the Creative Commons license and may not be reproduced without the prior and express written consent of Rice University.

  • Pre-algebra lessons
  • Pre-algebra word problems
  • Algebra lessons
  • Algebra word problems
  • Algebra proofs
  • Advanced algebra
  • Geometry lessons
  • Geometry word problems
  • Geometry proofs
  • Trigonometry lessons
  • Consumer math
  • Baseball math
  • Math for nurses
  • Statistics made easy
  • High school physics
  • Basic mathematics store
  • SAT Math Prep
  • Math skills by grade level
  • Ask an expert
  • Other websites
  • K-12 worksheets
  • Worksheets generator
  • Algebra worksheets
  • Geometry worksheets
  • Free math problem solver
  • Pre-algebra calculators
  • Algebra Calculators
  • Geometry Calculators
  • Math puzzles
  • Math tricks
  • Member login

Kinetic energy problems

When solving kinetic energy problems, you may be asked to find 3 variables. These variables are the kinetic energy, the mass, or the speed.

Problem # 1:

Suppose a car has 3000 Joules of kinetic energy. What will be its kinetic energy if the speed is doubled? What if the speed is tripled?

We already proved in kinetic energy lesson that whenever the speed is doubled, the kinetic energy is quadrupled or four times as big.

4 × 3000 = 12000

Therefore, the kinetic energy is going to be 12000 joules.

Let v be the speed of a moving object. Let speed =  3v after the speed is tripled. 

9 × 3000 = 27000

Therefore, the kinetic energy is going to be 27000 joules.

Problem # 2:

Calculate the kinetic energy of a 10 kg object moving with a speed of 5 m/s. Calculate the kinetic energy again when the speed is doubled.

Tricky kinetic energy problems

Problem # 3: 

Suppose a rat and a rhino are running with the same kinetic energy. Which one do you think is going faster?

The only tricky and hard part is to use the kinetic energy formula to solve for v.

Multiply both sides by 2

Problem # 4: 

The kinetic energy of an object is 8 times bigger than the mass. Is it possible to get speed of the object?

Think carefully and try to solve this problem yourself.

Potential energy

Recent Articles

How to divide any number by 5 in 2 seconds.

Feb 28, 24 11:07 AM

Math Trick to Square Numbers from 50 to 59

Feb 23, 24 04:46 AM

Sum of Consecutive Odd Numbers

Feb 22, 24 10:07 AM

100 Tough Algebra Word Problems. If you can solve these problems with no help, you must be a genius!

Math quizzes

 Recommended

About me :: Privacy policy :: Disclaimer :: Donate   Careers in mathematics  

Copyright © 2008-2021. Basic-mathematics.com. All right reserved

  • Physics Class-11th Notes
  • Physics Formulas
  • Physics Symbol
  • Application of Physics
  • Class 8 Science
  • Class 9 Science
  • Class 10 Science
  • Class 11 Science
  • Class 12 Science
  • Class 8 Study Material
  • Class 9 Study Material
  • Class 10 Study Material
  • Class 11 Study Material
  • Class 12 Study Material

Practice Problems on Kinetic Energy

  • Practice Problems on Potential Energy
  • Rotational Kinetic Energy Formula
  • Kinetic Energy and Molecular Speeds

Kinetic Energy

  • Real Life Applications of Kinetic Energy
  • Kinetic Theory of Gases
  • Solving Problems in Mechanics
  • Internal Energy Formula
  • Difference Between Kinetics and Kinematics
  • Mechanical Energy Formula
  • Sample Problems on Equation of Motion
  • Heat, Internal Energy and Work
  • Define Kinetic Energy, give two Examples of Kinetic Energy.
  • Program to calculate Kinetic Energy and Potential Energy
  • Minimum Initial Energy Required To Cross Street
  • Problem on Time Speed and Distance
  • Oracle Interview | Set 12 (On Campus for Application Dev Profile)
  • Sprinklr Interview Experience
  • Interview Experience Opera Solutions, Noida

When work is done by a force on an object. It acquires energy, it can be any form. Energy can take on many forms and can be converted from one form to another form. Potential energy, electric potential energy, kinetic energy, etc. are some examples of different types of energy. Kinetic energy comes when the object starts moving. This energy is due to motion. Although this energy is due to motion, this energy is not created. It is usually converted from one type of energy to another type. Let’s look at this concept in detail. 

If an object is stationary, and we want to put that object into motion. We need to apply force. Any type of acceleration requires some force. When this force is applied, work is done on the object. When the work is done on an object, this means energy is getting transferred to the object is one form or another. Force can be removed once the object is in motion, but till the time force was applied on the object. The work that was done during that time is converted into energy. 

Kinetic energy is the energy an object acquires by virtue of its motion. 

This energy can be transferred from one object to another. For example, a moving ball hitting a stationary ball might cause the other ball to move. In this situation, some kinetic energy of the ball is transferred to another ball. 

Formula of Kinetic Energy 

To calculate the kinetic energy of the object, let’s consider a scenario where a force F, is acting on an object of mass M. In this case, the object starts moving with the acceleration “a” and covers a distance of “d”. 

Work done in this case will be, 

The acceleration “a” can be replaced using an equation of motion. 

v 2 = u 2 + 2a.d

⇒v 2 – u 2 = 2a.d

⇒ [Tex]\frac{v^2 – u^2}{2a}[/Tex]  = d

Substituting the value of “d” in the equation, 

⇒ W =  [Tex]m.d.\frac{v^2 – u^2}{2d}[/Tex]

⇒W =  [Tex]m.\frac{v^2 – u^2}{2}[/Tex]

⇒W =  [Tex]\frac{1}{2}m(v^2 – u^2)[/Tex]

So, this whole work done is converted into the K.E of the object. 

In case, initial velocity u = 0, 

K.E =  [Tex]\frac{1}{2}mv^2[/Tex]

One can also say, the network work done on the system is equal to the change in kinetic energy of the object. 

Note:  1. Kinetic energy depends on the velocity of the object squared. This means, when th velocity of the object is doubled, its kinetic energy becomes four times.  2. K.E must always have zero or positive values.  3. Kinetic energy is a scalar quantity, and it is expressed in Joules. 

Sample Problems

Question 1: A ball has a mass of 2Kg, suppose it travels at 10m/s. Find the kinetic energy possessed by it. 

Answer: 

Given: m = 2Kg, and v = 10m/s  The KE is given by,  K.E =  [Tex]\frac{1}{2}mv^2[/Tex] K.E =  [Tex]\frac{1}{2}mv^2[/Tex] ⇒ K.E =  [Tex]\frac{1}{2}(2)(10)^2[/Tex] ⇒ K.E = 100J

Question 2: A ball has a mass of 10Kg, suppose it travels at 100m/s. Find the kinetic energy possessed by it. 

Given: m = 10Kg, and v = 100m/s  The KE is given by,  K.E =  [Tex]\frac{1}{2}mv^2[/Tex] K.E =  [Tex]\frac{1}{2}mv^2[/Tex] ⇒ K.E =  [Tex]\frac{1}{2}(10)(100)^2[/Tex] ⇒ K.E = 50000J

Question 3: A spaceship has a mass of 20000Kg, suppose it travels at 10m/s. Find the kinetic energy possessed by it. 

Given: m = 20000Kg, and v = 10m/s  The KE is given by,  K.E =  [Tex]\frac{1}{2}mv^2[/Tex] K.E =  [Tex]\frac{1}{2}mv^2[/Tex] ⇒ K.E =  [Tex]\frac{1}{2}(20000)(10)^2[/Tex] ⇒ K.E = 10 6 J

Question 4: Work done by a force on a moving object is 100J. It was traveling at a speed of 2 m/s. Find the new speed of the object if the mass of the object is 2Kg.

Answer:  Given: W = 100J Work done by the force is equal to the change in kinetic energy.  W =  [Tex]\frac{1}{2}m(v^2 – u^2)[/Tex] Given, u = 2 m/s and v = ?, m = 2kg.  Plugging the values in the given equation,  W =  [Tex]\frac{1}{2}m(v^2 – u^2)[/Tex] ⇒  [Tex]100 = \frac{1}{2}(2)(v^2 – 2^2)[/Tex] ⇒ [Tex]100 = v^2 – 2^2 \\ = 104 = v^2 \\ = v = \sqrt{104}  \text{ m/s}[/Tex] [Tex]v = \sqrt{104} \text{ m/s}[/Tex]

Question 5: Work done by a force on a moving object is -50J. It was traveling at a speed of 10m/s. Find the new speed of the object if the mass of the object is 2Kg.

Given: W = -50J Work done by the force is equal to the change in kinetic energy.  W =  [Tex]\frac{1}{2}m(v^2 – u^2)[/Tex] Given, u = 10m/s and v = ? . m = 2kg.  Plugging the values in the given equation,  W =  [Tex]\frac{1}{2}m(v^2 – u^2)[/Tex] ⇒  [Tex]-50 = \frac{1}{2}(2)(v^2 – 10^2)[/Tex] ⇒  [Tex]-50 = v^2 – 10^2 \\ = 50 = v^2 \\ = v = \sqrt{50}  \\ = v = 5\sqrt{2} \text{ m/s}[/Tex] The speed is decreased because the work done was negative. This means that the force was acting opposite to the block and velocity was decreased. 

Question 6: Suppose a 1000Kg was traveling at a speed of 10m/s. Now, this mass transfers all its energy to a mass of 10Kg. What will be the velocity of the 10Kg mass after being hit by it? 

KE is given by the formula, K.E =  [Tex]\frac{1}{2}mv^2     [/Tex]   KE of the heavier object  M = 1000Kg and v = 10m/s  K.E =  [Tex]\frac{1}{2}mv^2     [/Tex]   ⇒ K.E =  [Tex]\frac{1}{2}(1000)(10)^2[/Tex] ⇒K.E = 50,000J  Now this energy is transferred to another ball.  m = 10Kg and v = ? 50,000 =  [Tex]\frac{1}{2}(10)v^2[/Tex] ⇒ 10,000 = v 2 ⇒ v = 100 m/s

Question 7: Suppose a 10Kg mass was traveling at a speed of 100m/s. Now, this mass transfers all its energy to a mass of 20Kg. What will be the velocity of the 20Kg mass after being hit by it? 

To find the velocity of the 20 kg mass after being hit by the 10 kg mass, we use the principle of conservation of momentum. According to the conservation of momentum, the total momentum before the collision is equal to the total momentum after the collision, assuming no external forces are acting on the system. m 1 = 10kg v 1 = 100m/s m 2 =20kg v 2 = ? The total momentum before the collision (P initial) is : = m 1 x v 1 = 10 x 100 = 1000 Kgm/s According to the conservation of momentum: P initial = P final m 1 x v 1 = m 2 x v 2 10 x 100 = 20 x v 2 1000 = 20 x v 2 v 2 = 1000/20 = 50m/s So, the velocity of the 20 kg mass after being hit by the 10 kg mass is 50m/s.

Question 8: Suppose a 10Kg block was kept at 20m height. Now, this block is dropped. Find out the velocity of the block just before it hits the ground.

The block of 10Kg is kept at a height of 20m.  The potential energy of the block will be,  P.E = mgh  Here m = 10, g = 10m/s 2 and h = 20m.  P.E = mgh  ⇒ P.E = (10)(10)(20)  ⇒ P.E = 2000J  Now, this energy is converted completely into KE.  KE = PE  ⇒2000 =  [Tex]\frac{1}{2}mv^2[/Tex] Given m = 10Kg,  ⇒2000 =  [Tex]\frac{1}{2}10v^2[/Tex] ⇒400 = v 2 v = 20m/s 

Question 9: Suppose a rock of 100Kg was kept at 80m height. Now, this block is dropped. Find out the velocity of the block just before it hits the ground.

The block of 10Kg is kept at a height of 80m.  The potential energy of the block will be,  P.E = mgh  Here m = 100, g = 10m/s 2 and h = 80m.  P.E = mgh  ⇒ P.E = (100)(10)(80)  ⇒ P.E = 80000J  Now, this energy is converted completely into KE.  KE = PE  ⇒80000 =  [Tex]\frac{1}{2}mv^2[/Tex] Given m = 100Kg,  ⇒80000 =  [Tex]\frac{1}{2}100v^2[/Tex] ⇒1600 = v 2 v = 40m/s 

Please Login to comment...

Similar reads.

  • Physics-Class-11
  • School Learning
  • School Physics

Improve your Coding Skills with Practice

 alt=

What kind of Experience do you want to share?

HIGH SCHOOL

  • ACT Tutoring
  • SAT Tutoring
  • PSAT Tutoring
  • ASPIRE Tutoring
  • SHSAT Tutoring
  • STAAR Tutoring

GRADUATE SCHOOL

  • MCAT Tutoring
  • GRE Tutoring
  • LSAT Tutoring
  • GMAT Tutoring
  • AIMS Tutoring
  • HSPT Tutoring
  • ISAT Tutoring
  • SSAT Tutoring

Search 50+ Tests

Loading Page

math tutoring

  • Elementary Math
  • Pre-Calculus
  • Trigonometry

science tutoring

Foreign languages.

  • Mandarin Chinese

elementary tutoring

  • Computer Science

Search 350+ Subjects

  • Video Overview
  • Tutor Selection Process
  • Online Tutoring
  • Mobile Tutoring
  • Instant Tutoring
  • How We Operate
  • Our Guarantee
  • Impact of Tutoring
  • Reviews & Testimonials
  • About Varsity Tutors

High School Physics : Calculating Potential Energy

Study concepts, example questions & explanations for high school physics, all high school physics resources, example questions, example question #1 : calculating potential energy.

sample problem solving in potential and kinetic energy

At the top of the hill the skier has purely potential energy. At the bottom, she has purely kinetic energy. 

We can solve by understanding the conservation of energy. The skier's energy at the top of the hill will be equal to her energy at the bottom of the hill.

sample problem solving in potential and kinetic energy

Using the equations for potential and kinetic energy, we can solve for the height of the hill.

sample problem solving in potential and kinetic energy

The masses cancel, and we can plug in our final velocity and gravitational acceleration.

sample problem solving in potential and kinetic energy

This formula solves for the change in height. The negative sign implies she travelled in a downward direction. Because the question is asking how tall the hill is, we use an absolute value.

Example Question #2 : Calculating Potential Energy

sample problem solving in potential and kinetic energy

We can use conservation of energy to solve. The potential energy when the astronaut is holding the rock will be equal to the kinetic energy just before it strikes the surface.

sample problem solving in potential and kinetic energy

Plug in the given values and solve.

sample problem solving in potential and kinetic energy

Plug in the values, and solve for the potential energy.

sample problem solving in potential and kinetic energy

The units for energy are Joules.

Example Question #4 : Calculating Potential Energy

sample problem solving in potential and kinetic energy

Use the conservation of energy equation to solve for the potential energy at the top of the hill.

sample problem solving in potential and kinetic energy

Plug in the values given to you and solve for the final height.

sample problem solving in potential and kinetic energy

The maximum height will be when the ball has only potential energy, and no kinetic energy. Initially, the ball has only kinetic energy and no potential energy. We can set these values equal to each other due to conservation of energy.

sample problem solving in potential and kinetic energy

The masses will cancel out.

sample problem solving in potential and kinetic energy

Plug in the values that were given and solve for the height.

sample problem solving in potential and kinetic energy

We are given the height and mass of the ball. Using the given values, we can solve for the potential energy.

Keep in mind that the displacement will be negative because the ball is traveling in the downward direction.

sample problem solving in potential and kinetic energy

Since we know the mass, height, and acceleration from gravity, we can simply multiply to find the potential energy.

sample problem solving in potential and kinetic energy

Example Question #9 : Calculating Potential Energy

sample problem solving in potential and kinetic energy

Given the values for the mass, height, and gravity, we can solve using multiplication. Note that the height is negative because the book falls in the downward direction.

sample problem solving in potential and kinetic energy

Report an issue with this question

If you've found an issue with this question, please let us know. With the help of the community we can continue to improve our educational resources.

DMCA Complaint

If you believe that content available by means of the Website (as defined in our Terms of Service) infringes one or more of your copyrights, please notify us by providing a written notice (“Infringement Notice”) containing the information described below to the designated agent listed below. If Varsity Tutors takes action in response to an Infringement Notice, it will make a good faith attempt to contact the party that made such content available by means of the most recent email address, if any, provided by such party to Varsity Tutors.

Your Infringement Notice may be forwarded to the party that made the content available or to third parties such as ChillingEffects.org.

Please be advised that you will be liable for damages (including costs and attorneys’ fees) if you materially misrepresent that a product or activity is infringing your copyrights. Thus, if you are not sure content located on or linked-to by the Website infringes your copyright, you should consider first contacting an attorney.

Please follow these steps to file a notice:

You must include the following:

A physical or electronic signature of the copyright owner or a person authorized to act on their behalf; An identification of the copyright claimed to have been infringed; A description of the nature and exact location of the content that you claim to infringe your copyright, in \ sufficient detail to permit Varsity Tutors to find and positively identify that content; for example we require a link to the specific question (not just the name of the question) that contains the content and a description of which specific portion of the question – an image, a link, the text, etc – your complaint refers to; Your name, address, telephone number and email address; and A statement by you: (a) that you believe in good faith that the use of the content that you claim to infringe your copyright is not authorized by law, or by the copyright owner or such owner’s agent; (b) that all of the information contained in your Infringement Notice is accurate, and (c) under penalty of perjury, that you are either the copyright owner or a person authorized to act on their behalf.

Send your complaint to our designated agent at:

Charles Cohn Varsity Tutors LLC 101 S. Hanley Rd, Suite 300 St. Louis, MO 63105

Or fill out the form below:

Contact Information

Complaint details.

Learning Tools by Varsity Tutors

Email address:
Your name:
Feedback:

If you're seeing this message, it means we're having trouble loading external resources on our website.

If you're behind a web filter, please make sure that the domains *.kastatic.org and *.kasandbox.org are unblocked.

To log in and use all the features of Khan Academy, please enable JavaScript in your browser.

Middle school physics

Course: middle school physics   >   unit 3, potential energy.

  • Understand: potential energy

sample problem solving in potential and kinetic energy

Key points:

  • Potential energy is energy that has the potential to become another form of energy . An object’s potential energy depends on its physical properties and position in a system.
  • Potential energy comes in many forms, such as:
Gravitational potential energy due to an object’s mass and position in a gravitational field Magnetic potential energy due to a magnetic object’s position in a magnetic field Electric potential energy due to the size of an electric charge and its position in an electric field Elastic potential energy of a spring or rubber band that is stretched
  • Changing an object’s position can change its potential energy. This depends on the forces between objects.
For example, if two objects attract each other, moving them apart will increase their potential energies. If two objects repel each other, moving them apart will decrease their potential energies.

Want to join the conversation?

  • Upvote Button navigates to signup page
  • Downvote Button navigates to signup page
  • Flag Button navigates to signup page

Incredible Answer

Library homepage

  • school Campus Bookshelves
  • menu_book Bookshelves
  • perm_media Learning Objects
  • login Login
  • how_to_reg Request Instructor Account
  • hub Instructor Commons

Margin Size

  • Download Page (PDF)
  • Download Full Book (PDF)
  • Periodic Table
  • Physics Constants
  • Scientific Calculator
  • Reference & Cite
  • Tools expand_more
  • Readability

selected template will load here

This action is not available.

Physics LibreTexts

8.8: Sample problems and solutions

  • Last updated
  • Save as PDF
  • Page ID 19419

\( \newcommand{\vecs}[1]{\overset { \scriptstyle \rightharpoonup} {\mathbf{#1}} } \)

\( \newcommand{\vecd}[1]{\overset{-\!-\!\rightharpoonup}{\vphantom{a}\smash {#1}}} \)

\( \newcommand{\id}{\mathrm{id}}\) \( \newcommand{\Span}{\mathrm{span}}\)

( \newcommand{\kernel}{\mathrm{null}\,}\) \( \newcommand{\range}{\mathrm{range}\,}\)

\( \newcommand{\RealPart}{\mathrm{Re}}\) \( \newcommand{\ImaginaryPart}{\mathrm{Im}}\)

\( \newcommand{\Argument}{\mathrm{Arg}}\) \( \newcommand{\norm}[1]{\| #1 \|}\)

\( \newcommand{\inner}[2]{\langle #1, #2 \rangle}\)

\( \newcommand{\Span}{\mathrm{span}}\)

\( \newcommand{\id}{\mathrm{id}}\)

\( \newcommand{\kernel}{\mathrm{null}\,}\)

\( \newcommand{\range}{\mathrm{range}\,}\)

\( \newcommand{\RealPart}{\mathrm{Re}}\)

\( \newcommand{\ImaginaryPart}{\mathrm{Im}}\)

\( \newcommand{\Argument}{\mathrm{Arg}}\)

\( \newcommand{\norm}[1]{\| #1 \|}\)

\( \newcommand{\Span}{\mathrm{span}}\) \( \newcommand{\AA}{\unicode[.8,0]{x212B}}\)

\( \newcommand{\vectorA}[1]{\vec{#1}}      % arrow\)

\( \newcommand{\vectorAt}[1]{\vec{\text{#1}}}      % arrow\)

\( \newcommand{\vectorB}[1]{\overset { \scriptstyle \rightharpoonup} {\mathbf{#1}} } \)

\( \newcommand{\vectorC}[1]{\textbf{#1}} \)

\( \newcommand{\vectorD}[1]{\overrightarrow{#1}} \)

\( \newcommand{\vectorDt}[1]{\overrightarrow{\text{#1}}} \)

\( \newcommand{\vectE}[1]{\overset{-\!-\!\rightharpoonup}{\vphantom{a}\smash{\mathbf {#1}}}} \)

Exercise \(\PageIndex{1}\)

A ball of mass \(m\) is dropped onto a vertical spring with spring constant \(k\) . The spring will compress until the ball comes to rest. How much will it compress if the ball is dropped from a height \(h\) above the spring?

clipboard_eab829ec0a7492aa08816c0bd27d5a806.png

The two forces acting on the ball are gravity and the spring force. Both are conservative, so we can use conservation of mechanical energy. We will find the energy of the ball when it is at a height \(h\) above the spring, and the energy of the ball when the spring is fully compressed. Then, we will use conservation of mechanical energy to determine the compression of the spring.

Remember that the total mechanical energy is the sum of the total potential energy and the kinetic energy, \(E=U+K\) . Let’s call the initial position of the ball \(A\) and the final position of the ball \(B\) . You will notice that we set up our coordinate system so that \(y\) is positive upwards, with \(y=0\) at the point where the ball comes into contact with the spring. We choose to define both the gravitational potential energy and spring potential energy so that they are zero at \(y=0\) .

Since the ball starts from rest, its kinetic energy is zero at position \(A\) . At this point, the ball is not touching the spring, so the potential energy from the spring force is zero. The mechanical energy of the ball at position \(A\) is simply equal to its gravitational potential energy:

\[\begin{aligned} E_A&=U_A+K_A\\[4pt] E_A&=mgh\end{aligned}\]

At position \(B\) , the ball is again at rest, so the kinetic energy of the ball is zero. Now that the ball is in contact with the spring, it will experience a force from the spring that can be modeled with a potential energy \(U(y)=\frac{1}{2}ky_1^2\) , where \(y_1\) is the distance between the rest position of the spring and its compressed length. At point \(B\) ( \(y=-y_1\) ), the ball will have both spring and gravitational potential energy, so its mechanical energy at position \(B\) is given by:

\[\begin{aligned} E_B&=U_B+K_B=U_B\\[4pt] U_B&=mg(-y_1)+\frac{1}{2}ky_1^2\\[4pt] E_B&=-mgy_1+\frac{1}{2}ky_1^2\end{aligned}\]

Since mechanical energy is conserved in this system (no non-conservative forces are doing work), we can now set \(E_A=E_B\) and solve for \(y_1\) :

\[\begin{aligned} E_A&=E_B\\[4pt] mgh&=-mgy_1+\frac{1}{2}ky_1^2\\[4pt] 0&=\frac{1}{2}ky_1^2-mgy_1-mgh\\[4pt]\end{aligned}\]

where in the last line we rewrote the expression as a quadratic equation. We can solve for \(y_1\) with the quadratic formula:

\[\begin{aligned} y_1=\frac{mg\pm\sqrt{(mg)^2-4(1/2k)(-mgh)}}{k}\\[4pt] y_1=\frac{mg\pm\sqrt{mg(mg+2kh)}}{k}\end{aligned}\]

We now have an expression for the amount the spring is compressed, \(y_1\) , in terms of our known values.

Exercise \(\PageIndex{2}\)

A simple pendulum consists of a mass \(m\) connected to a string of length \(L\) . The pendulum is released from an angle \(\theta_0\) from the vertical. Use conservation of energy to find an expression for the velocity of the mass as a function of the angle.

clipboard_e54af25593c52c2fc2ba8f557d3f46923.png

We are going to find a general expression for the energy of the system, and then use this expression to find the velocity at any point. There are two forces acting on the mass:

The force of tension (from the string). This force is perpendicular to the direction of motion at any point, so it does no work on the mass.

The force of gravity, which has a potential energy function given by \(U(y)=mgy\) . We choose the gravitational potential energy to be zero when the pendulum hangs vertically (when \(\theta=0\) and \(y=0\) ).

The mechanical energy of the mass is conserved, and at any point is given by the sum of its kinetic and its gravitational potential energies:

\[\begin{aligned} E=mgy+\frac{1}{2}mv^2\end{aligned}\]

We want to find the velocity as a function of \(\theta\) , so we need to write \(y\) in terms of \(\theta\) . As you may recall from Problem 7.6.2 , we saw that from the geometry of the problem, we can express the height of the mass as \(y=L-L\cos\theta\) , or \(L(1-\cos\theta)\) , where \(y\) is the height as measured from the bottom point of the motion. You can refer to Figure 7.6.4 to refresh your memory. The energy at any point is then:

\[\begin{aligned} E=mgL(1-\cos\theta)+\frac{1}{2}mv^2\end{aligned}\]

Conservation of energy tells us that the total energy at any point must be the same as the initial energy. So, we can use our initial conditions to find the total energy of the system. The mass starts from rest (initial kinetic energy is zero) an angle \(\theta_0\) above the vertical:

\[\begin{aligned} E&=mgL(1-\cos\theta)+\frac{1}{2}mv^2\\[4pt] E_{initial}&=mgL(1-\cos\theta_0)\end{aligned}\]

Now that we have found the total energy of the system, we can write our general expression for the energy of the system at any point:

\[\begin{aligned} E&=mgL(1-\cos\theta)+\frac{1}{2}mv^2\\[4pt] mgL(1-\cos\theta_0)&=mgL(1-\cos\theta)+\frac{1}{2}mv^2\end{aligned}\]

All that’s left to do is simplify the expression and rearrange for \(v\) :

\[\begin{aligned} mgL(1-\cos\theta_0)&=mgL(1-\cos\theta)+\frac{1}{2}mv^2\\[4pt] gL(1-\cos\theta_0)-gL(1-\cos\theta)&=\frac{1}{2}v^2\\[4pt] gL-gL\cos\theta_0-gL+gL\cos\theta&=\frac{1}{2}v^2\\[4pt] gL(\cos\theta-\cos\theta_0)&=\frac{1}{2}v^2\\[4pt] \therefore v&=\sqrt{2gl(\cos\theta-\cos\theta_0)}\end{aligned}\]

We can see from this expression that the speed will be maximized when \(\cos\theta\) is maximized, which will occur when \(\theta=0\) (when the pendulum is vertical). This is as we expected. We can also see that we will get an imaginary number if the magnitude of \(\theta\) is greater than \(\theta_0\) , showing that the motion is constrained between \(-\theta_0\) and \(\theta_0\) . Finally, we showed that the velocity of the pendulum does not depend on the mass!

Exercise \(\PageIndex{3}\)

A block of mass \(m\) sits on a frictionless horizontal surface. It is attached to a wall by a spring with a spring constant \(k\) . The mass is pushed so as to compress the spring and then it is released (Figure \(\PageIndex{3}\)). Use the Lagrangian formalism to find an equation of motion for the mass/spring system (i.e. use the Lagrangian to determine the acceleration of the mass).

clipboard_e7fbdd6483286995bd45d54b49523b0b9.png

We are going to find an equation of motion of the system using the Lagrangian method. We choose to use a one dimension coordinate system, with the \(x\) axis defined to be co-linear with the spring, positive in the direction where the spring is extended, and set the origin to be located at the rest position of the spring. The kinetic energy and potential energy of the mass are given by

\[\begin{aligned} K&=\frac{1}{2}mv_x^2\\[4pt] U&=\frac{1}{2}kx^2\end{aligned}\]

since the only force exerted on the mass that can do work is the force from the spring. We have chosen the potential energy to be zero at \(x=0\) . The Lagrangian for this system is:

\[\begin{aligned} L&=K-U\\[4pt] L&=\frac{1}{2}mv_x^2-\frac{1}{2}kx^2\end{aligned}\]

The Euler-Lagrange equation in one dimension is:

\[\begin{aligned} \frac{d}{dt}\left(\frac{\partial L}{\partial v_{x}}\right)-\frac{\partial L}{\partial x} = 0\end{aligned}\]

We can calculate the terms of the Euler-Lagrange equation:

\[\begin{aligned} \frac{\partial L}{\partial v_{x}}&=\frac{\partial}{\partial v_{x}}\left(\frac{1}{2}mv_x^2-\frac{1}{2}kx^2\right)\\[4pt] &=mv_x\\[4pt] \therefore \frac{d}{dt}\left(\frac{\partial L}{\partial v_{x}}\right)&=\frac{d}{dt}(mv_x)\\[4pt] &=ma_x\\[4pt] \textrm{and}\qquad \frac{\partial L}{\partial x}&=\left(\frac{1}{2}mv_x^2-\frac{1}{2}kx^2\right)\\[4pt] &=-kx\end{aligned}\]

and then put them together to get:

\[\begin{aligned} \frac{d}{dt}\left(\frac{\partial L}{\partial v_{x}}\right)-\frac{\partial L}{\partial x} &= 0\\[4pt] \therefore ma_x&=-kx\\[4pt]\end{aligned}\]

We can see that this equation of motion is equivalent to Newton’s Second Law.

logo

Have an account?

pencil-icon

Potential and Kinetic Energy Problems

9th - 12th grade.

User image

10 questions

Player avatar

Introducing new   Paper mode

No student devices needed.   Know more

The kinetic energy of a 7 kg cat moving at 4 m/s is

The gravitational potential energy of an object on the ground is

equal to the kinetic energy

The potential energy of a 550 kg missile flying at 800 meters is

4,312,000 Joules

440,000 Joules

176,000,000 Joules

A 80 kg runner going at 2.8 m/s has a kinetic energy of

2195.2 Joules

313.6 Joules

627.2 Joules

A ball that has a mass of 3kg and has a potential energy of 5,000 J is how high in the air.

A soccer ball has a gravitational potential energy of 80%. What is the percent of kinetic energy?

A student raises a 0.3 kg phone to a height of 0.8 meters. The potential energy of the phone is

.096 Joules

2.35 Joules

0.24 Joules

A plane with a mass 1200kg and a kinetic energy of 3,500,000 J is flying with a velocity of ?

5,833.3 m/s

What is the mass of a football that feel from a height of 28m with a potential energy of 4,150J?

A cannonball is launched with 500 Joules of kinetic energy and a velocity of 10m/s. What is its mass?

Explore all questions with a free account

Google Logo

Continue with email

Continue with phone

easycalculation.com

Potential Energy Examples

sample problem solving in potential and kinetic energy

Potential energy (PE) is the energy that is stored in an object due to its position charge, stress etc. Here are a few potential energy examples with solutions. These potential energy practice problems will help you learn how to calculate PE, mass, height.

Potential Energy Practice Problems

Let us consider a potential energy practice problem : An object of mass 30 kgs is placed on a hill top of height 80m. What is the potential energy possessed by the object?

We can calculate the potential energy of the object using the given formula

Substituting the values in the formula,

PE = m x g x h      = 30 x 9.8 x 80      = 23520 J

Note: We know that the acceleration due to gravity is constant and is always equal to 9.8 m/s 2

Therefore, the potential energy of the object is 23520 J .

Refer the below potential energy sample problem and calculate mass based on the potential energy, height and gravity. A fruit hangs from a tree and is about to fall to the ground of 10 meters height. It has a potential energy of 22.5 J. Calculate the mass of the fruit

Substitute the values using the given formula, M = PE / (g x h)      = 22.5 / (9.8 x 10)      = 0.229 kg

Therefore, the value of Mass is 0.229 kg

Refer the below example of potential energy and learn how to calculate height from PE and mass.

Let us consider a potential energy worked example problem: 5 kg weighing cat climbing at the top of the tree has a potential energy of 1176 kg. Find the height of the tree?

Substitute the values using the given formula, H = PE / (m x g)      = 1176 / (9.8 x 5)      = 24 cm

Therefore, the height of the tree is 24 cm

Related Examples:

  • Young's Modulus Calculation Examples
  • Kinetic Energy Example
  • Force Examples
  • Work Physics Problems With Solutions
  • Newton Second Law Of Motion Example Problems With Answers
  • Synthetic Division Examples

Calculators and Converters

  • Classical Physics

Top Calculators

Popular calculators.

  • Derivative Calculator
  • Inverse of Matrix Calculator
  • Compound Interest Calculator
  • Pregnancy Calculator Online

Top Categories

Understanding Work, Energy & Momentum in Physics

IMAGES

  1. PPT

    sample problem solving in potential and kinetic energy

  2. Kinetic and Potential Energy Problem Solving

    sample problem solving in potential and kinetic energy

  3. Kinetic and Potential Energy Problems

    sample problem solving in potential and kinetic energy

  4. Physics Lesson

    sample problem solving in potential and kinetic energy

  5. Kinetic Energy sample problem #1

    sample problem solving in potential and kinetic energy

  6. Physics:2.1.4.1 Solving for the kinetic energy of an object using the equation

    sample problem solving in potential and kinetic energy

VIDEO

  1. Potential & Kinetic energy تجربة الطاقة الكامنة والحركية

  2. Dimensions of potential & Kinetic energy #kineticenergy #potentialenergy #Shortvidioes #class #Eraj

  3. Solving Kinetic Energy Sample Problem with Mrs. Aki

  4. Potential Energy Questions

  5. Numericals on Percentage Change in Kinetic Energy

  6. Work

COMMENTS

  1. Potential And Kinetic Energy Example Problem

    Solution: The total energy of the cart is expressed by the sum of its potential energy and its kinetic energy. Potential energy of an object in a gravitational field is expressed by the formula. PE = mgh. where. PE is the potential energy. m is the mass of the object. g is the acceleration due to gravity = 9.8 m/s 2.

  2. PDF Potential and Kinetic Energy Practice Problems

    Show all of your math when answering the problems below. Write directly on this page. 1. A 1 kg rock is at a height of 100 meters. a. What is the rock's gravitational potential energy at 100 meters high? b. Calculate the rock's gravitational potential energy at 50 m, 20 m, 1 m, and 0 m high. Put the answers in the data table below.

  3. Potential and Kinetic Energy

    Energy. Energy is the capacity to do work. The unit of energy is J (Joule) which is also kg m 2 /s 2 (kilogram meter squared per second squared) Energy can be in many forms! Here we look at Potential Energy (PE) and Kinetic Energy (KE). Potential Energy and Kinetic Energy . A hammer: when raised up has potential energy (the energy of position ...

  4. Mechanical Energy Problem Solutions

    I cancelled out the initial kinetic energy because: KE i = ½ mv f2. KE i = (½) (3.5) (0 2) = 0 J. I cancelled out the final potential energy because: PE f = mgh f. PE f = (3.5) (9.8) (0) = 0 J. (Note: In many of these problems I could cancel out mass but did not since it was provided) Since I did not cancel out mass I could answer the ...

  5. Work, Energy, and Power Problem Sets

    What is Jerome's kinetic energy at the end of this acceleration period? c. Once the launch is over, Jerome begins screaming up the 420-foot, completely vertical section of the track. Determine Jerome's potential energy at the top of the vertical section. (GIVEN: 1.00 m = 3.28 ft) d. Determine Jerome's kinetic energy at the top of the vertical ...

  6. Potential and Kinetic Energy with Practice Problems

    About Press Copyright Contact us Creators Advertise Developers Terms Privacy Policy & Safety How YouTube works Test new features NFL Sunday Ticket Press Copyright ...

  7. Lesson Kinetic and Potential Energy of Motion

    Explain the ideas of potential energy and kinetic energy as two different kinds of mechanical energy. Give definitions of each and present the equations, carefully explaining each variable, as discussed in the next section, PE = mass x g x height. and. KE = 1/2 m x v2. Explain how energy can be converted from one form to another.

  8. Work, Energy, and Power Problem Sets

    Practice Review Test Concept Builders. About the Concept Builders; Version 2; ... Use the work-energy theorem to calculate either the work or the kinetic energy. All problems are highly scaffolded. Includes 6 problems. Problem Set WE10: Work and Kinetic Energy 2 ... Some problems involve elastic potential energy. Includes 8 problems. Problem ...

  9. PDF Kinetic and Potential Energy Practice Problems

    Kinetic and Potential Energy Practice Problems Solve the following problems and show your work! 1. A car has a mass of 2,000 kg and is traveling at 28 meters per second. What is the car's kinetic energy? 2. When a golf ball is hit, it travels at 41 meters per second. The mass of a golf ball is 0.045 kg. What is the kinetic energy of the golf ...

  10. PDF Kinetic and Potential Energy

    2. Next, we place this information into the kinetic energy formula: • KE = 1/2 mv2. • KE = 1/2 (45 kg)(13 m/sec)2. 3. Solving the equation gives a kinetic energy value of 3802.5 J. Note: The unit for energy is the same as for work: the Joule (J). Sample Problem. The kinetic energy of a boat is calculated at 52,000 J.

  11. Kinetic and Potential Energy Problem Set

    Practice problems for physics students on potential energy and kinetic energy. These are very simple problems that can be solved without the use of a calculator. ... Find the gravitational potential energy of a light that has a mass of 13.0 kg and is 4.8 m above the ground. m = g = Answer: h = GPE = 2.

  12. Using the kinetic energy equation (practice)

    A cannon launches a 3.0 kg pumpkin with 110 J of kinetic energy. What is the pumpkin's speed? Round answer to two significant digits. Learn for free about math, art, computer programming, economics, physics, chemistry, biology, medicine, finance, history, and more. Khan Academy is a nonprofit with the mission of providing a free, world-class ...

  13. Practice Problems on Potential Energy

    Sample Problems. Question 1: A mass of 2Kg is taken from the ground to the height of 10m. Find the potential energy of the object. Answer: The potential energy of a mass 'm' at the height 'h' is given by, P = mgh. Given: m = 2kg and g = 10 m/s 2 and h = 10m. Aim: Find the potential energy.

  14. Practice Problems for Kinetic and Potential Energy Flashcards

    KE = 196 j. At what height is an object that weighs 490 N if its potential energy is 4900 N-m? 10 m. A motorbike has 4500 j of KE and is traveling at 15 m/s. What is its mass? 45 kg. Some practice with energy. Formulas - (Kinetic Energy) KE = (MV^2)/2 (Gravitational Potential Energy) GPE = WH (Weight) W = 9.8M (Mass) M = W/9.8 These….

  15. 7.2 Kinetic Energy and the Work-Energy Theorem

    Kinetic energy is a form of energy associated with the motion of a particle, single body, or system of objects moving together. We are aware that it takes energy to get an object, like a car or the package in Figure 7.4, up to speed, but it may be a bit surprising that kinetic energy is proportional to speed squared. This proportionality means ...

  16. Kinetic Energy Problems and Solutions

    Problem # 2: Calculate the kinetic energy of a 10 kg object moving with a speed of 5 m/s. Calculate the kinetic energy again when the speed is doubled. Solution: K = 5 × 25 = 125 joules. Now, calculate again whe the speed is doubled. K = 5 × 100 = 500 joules. As you can see, the kinetic energy is quadrupled since 4 × 125 = 500.

  17. Practice Problems on Kinetic Energy

    1. Kinetic energy depends on the velocity of the object squared. This means, when th velocity of the object is doubled, its kinetic energy becomes four times. 2. K.E must always have zero or positive values. 3. Kinetic energy is a scalar quantity, and it is expressed in Joules. Sample Problems.

  18. 6.5: Potential Energy and Conservation of Energy

    The work-energy theorem states that the net work done by all forces acting on a system equals its change in kinetic energy (KE). In equation form, this is: Wnet = 1 2mv2 − 1 2mv2 0 = ΔKE. If only conservative forces act, then Wnet = Wc, where W c is the total work done by all conservative forces. Thus, Wc = ΔKE.

  19. Calculating Potential Energy

    Correct answer: Explanation: The equation for potential energy is . We are given the mass of the ball, the height of the table, and the acceleration of gravity in the question. The distance the ball travels is in the downward direction, making it negative. Plug in the values, and solve for the potential energy.

  20. Potential energy (article)

    Key points: Potential energy is energy that has the potential to become another form of energy. An object's potential energy depends on its physical properties and position in a system. Potential energy comes in many forms, such as: Gravitational potential energy due to an object's mass and position in a gravitational field.

  21. 8.8: Sample problems and solutions

    The force of gravity, which has a potential energy function given by \(U(y)=mgy\). We choose the gravitational potential energy to be zero when the pendulum hangs vertically (when \(\theta=0\) and \(y=0\)). The mechanical energy of the mass is conserved, and at any point is given by the sum of its kinetic and its gravitational potential energies:

  22. Potential and Kinetic Energy Problems

    The kinetic energy of a 7 kg cat moving at 4 m/s is. 28 Joules. 56 Joules. 112 Joules. 2. Multiple Choice. 30 seconds. 1 pt. The gravitational potential energy of an object on the ground is.

  23. Potential Energy Examples

    Therefore, the potential energy of the object is 23520 J. Example 2: Refer the below potential energy sample problem and calculate mass based on the potential energy, height and gravity. A fruit hangs from a tree and is about to fall to the ground of 10 meters height. It has a potential energy of 22.5 J. Calculate the mass of the fruit. Solution:

  24. Understanding Work, Energy & Momentum in Physics

    of mechanical energy equation, and the equations for calculating potential energy and kinetic energy. 6. Pay attention to units: Make sure to use consistent units throughout your calculations and convert units if necessary. It's also important to pay attention to the units of the answer, and make sure it makes sense in the context of the problem. 7.

  25. Math Review Practice Problems

    A nuclear research institute wishes to have a collision between a 32 MeV (at impact) alpha particle and a stationary polonium-209. Determine the kinetic energy of the alpha particle at the instant it is fired toward the polonium nucleus. Take the polonium nuclei to be rest at all times, and do not treat the alpha particle like a point particle.